Đến nội dung

Hình ảnh

Topic về Bất đẳng thức, cực trị THCS


  • Please log in to reply
Chủ đề này có 1205 trả lời

#21
No Problem

No Problem

    Hạ sĩ

  • Thành viên
  • 67 Bài viết

Bài 6:(THCS) Cho $x,y,z>0$ và $x+y+z=1$. CMR: $x+2y+z\geq 4(x+y)(y+z)(z+x)$

Bài 7:(THCS) Với $a,b,c \in [0,1]$. Tìm GTLN của biểu thức: $A=\dfrac{a}{bc+1}+\dfrac{b}{ac+1}+\dfrac{c}{ab+1}$


Giả sử $0\le \ a\le \ b\le \ c\le \ 1$
:pe $bc+1\ge \ ac+1\ge \ ab+1$ và $(1-a)(1-b)\ge 0 $ :pe$ ab+1\ge \ a+b$
:Rightarrow $A\le \dfrac{a+b}{ab+1}+\dfrac{c}{ab+1}\le \ 1+c\le 2$
Đẳng thức xảy ra :D $(a;b;c)=(0;1;1)$ và các hoán vị
p/s:mình có ý kiến khi giải xong các bài thì nên nêu ra đk xảy ra dấu bằng :(

Bài viết đã được chỉnh sửa nội dung bởi No Problem: 03-07-2009 - 08:19


#22
No Problem

No Problem

    Hạ sĩ

  • Thành viên
  • 67 Bài viết

Bài 10:(THCS) Với $a,b,c$ là 3 cạnh của tam giác. CMR: $\dfrac{a}{\sqrt[3]{b^{3}+c^{3}}}+ \dfrac{b}{\sqrt[3]{a^{3}+c^{3}}}+ \dfrac{c}{\sqrt[3]{a^{3}+b^{3}}}<2\sqrt[3]{5}$

Các bài của THCS được ghi bên cạnh là chữ THCS, bài toán của THPT được ghi là THPT .
Rất mong các bạn post bài toán lên topic để mọi người cùng giải. Những công thức mình post có thể được dùng trực tiếp , nhưng khi đi thi thì ko thể được đâu mà phải chứng mình nó, trừ 2 BĐT là cô si và Bunhiacopxki.


$\dfrac{a}{\sqrt[3]{b^{3}+c^{3}}}\le \dfrac{a}{\sqrt[3]{\dfrac{(b+c)^3}{4}}}=\dfrac{\sqrt[3]{4}a}{b+c}$
mà $\sum\dfrac{a}{b+c}\le 2$
:Rightarrow $VT\le 2\sqrt[3]{4}$
p/s:LHS là cái gì vậy :( :D :pe :Rightarrow :pe :leluoi:

Bài viết đã được chỉnh sửa nội dung bởi No Problem: 03-07-2009 - 08:29


#23
No Problem

No Problem

    Hạ sĩ

  • Thành viên
  • 67 Bài viết
Bài 12[b] Cho$a,b,c>0$.ch/m:
$\dfrac{a^2(b+c)}{b^2+c^2}+\dfrac{b^2(c+a)}{c^2+a^2}+\dfrac{c^2(a+b)}{a^2+b^2}\ge a+b+c$
[b]Bài 13
Cho $a,b,c>0;ab+bc+ca=1$.Ch/m:
$\dfrac{1}{ab}+\dfrac{1}{bc}+\dfrac{1}{ca}\ge \ 3+\sqrt{1+\dfrac{1}{a^2}}+\sqrt{1+\dfrac{1}{b^2}}+\sqrt{1+\dfrac{1}{c^2}}$

#24
cvp

cvp

    Sĩ quan

  • Thành viên
  • 400 Bài viết

Bài 13 Cho $a,b,c>0;ab+bc+ca=1$.Ch/m:
$\dfrac{1}{ab}+\dfrac{1}{bc}+\dfrac{1}{ca}\ge \ 3+\sqrt{1+\dfrac{1}{a^2}}+\sqrt{1+\dfrac{1}{b^2}}+\sqrt{1+\dfrac{1}{c^2}}$

Dùng giả thiết $ab+bc+ca=1$
BĐT $<=>3+\dfrac{c(a+b)}{ab}+\dfrac{b(c+a)}{ca}+\dfrac{a(b+c)}{bc} \ge 3+\dfrac{\sqrt{(a+b)(a+c)}}{a}+\dfrac{\sqrt{(b+a)(b+c)}}{b}+\dfrac{\sqrt{(c+a)(c+b)}}{c}$
Chứng minh $\dfrac{c(a+b)}{ab}+\dfrac{b(c+a)}{ca}+\dfrac{a(b+c)}{bc}\ge \dfrac{\sqrt{(a+b)(a+c)}}{a}+\dfrac{\sqrt{(b+a)(b+c)}}{b}+\dfrac{\sqrt{(c+a)(c+b)}}{c}$
Đến đây dùng AM-GM: $\dfrac{c(a+b)}{ab}+\dfrac{b(c+a)}{ca}\ge 2\dfrac{\sqrt{(a+b)(a+c)}}{a}$
Tương tự => đpcm
Dấu = khi $a=b=c=\dfrac{1}{\sqrt{3}}$

Bài viết đã được chỉnh sửa nội dung bởi cvp: 03-07-2009 - 09:22

Hình đã gửi


#25
cvp

cvp

    Sĩ quan

  • Thành viên
  • 400 Bài viết
Mình đóng góp bài nè:
Bài 14: Cho a,b,c dương thỏa mãn abc=1. Chứng minh rằng:
$\dfrac{1}{b(5a+b)}+\dfrac{1}{c(5b+c)}+\dfrac{1}{a(5c+a)}\ge \dfrac{1}{2}$

Bài viết đã được chỉnh sửa nội dung bởi cvp: 03-07-2009 - 10:15

Hình đã gửi


#26
No Problem

No Problem

    Hạ sĩ

  • Thành viên
  • 67 Bài viết

Mình đóng góp bài nè:
Bài 14: Cho a,b,c dương thỏa mãn abc=1. Chứng minh rằng:
$\dfrac{1}{b(5a+b)}+\dfrac{1}{c(5b+c)}+\dfrac{1}{a(5c+a)}\ge \dfrac{1}{2}$


Đặt $a=\dfrac{x}{y},b=\dfrac{y}{z},c=\dfrac{z}{x}$
$Vt=\dfrac{x^2}{z^2+5xy}+\dfrac{y^2}{x^2+5yz}+\dfrac{z^2}{y^2+5xz}\ge \dfrac{(x+y+z)^2}{(x+y+z)^2+3(xy+yz+zx)}\ge \dfrac{1}{2}$
Đẳng thức xảy ra :neq a=b=c=1 :)

#27
cvp

cvp

    Sĩ quan

  • Thành viên
  • 400 Bài viết
Tiếp một bài dành cho THCS
Bài 15Cho $x,y,z$ dương.Chứng minh rằng :
$\dfrac{1}{x^2+yz}+\dfrac{1}{y^2+zx}+\dfrac{1}{z^2+xy}\le \dfrac{1}{2}(\dfrac{1}{xy}+\dfrac{1}{yz}+\dfrac{1}{zx}$

Hình đã gửi


#28
Hero Math

Hero Math

    Anh hùng của diễn đàn .

  • Thành viên
  • 237 Bài viết

Tiếp một bài dành cho THCS
Bài 15Cho $x,y,z$ dương.Chứng minh rằng :
$\dfrac{1}{x^2+yz}+\dfrac{1}{y^2+zx}+\dfrac{1}{z^2+xy}\le \dfrac{1}{2}(\dfrac{1}{xy}+\dfrac{1}{yz}+\dfrac{1}{zx}$

Cảm ơn anh Dũng và Minh đã đóng góp các bài tập.Áp dụng BĐT Cô si và BĐT: $ab+bc+ac\leq a^{2}+b^{2}+c^{2}. $ta có:
$VT=\dfrac{1}{x^2+yz}+\dfrac{1}{y^2+zx}+\dfrac{1}{z^2+xy}\leq \dfrac{1}{2}(\dfrac{1}{x\sqrt{yz}}+\dfrac{1}{y\sqrt{xz}}+\dfrac{1}{z\sqrt{xy}})$
$=\dfrac{1}{2}.\dfrac{\sqrt{xy}+\sqrt{yz}+\sqrt{xz}}{xyz}\leq \dfrac{1}{2}.\dfrac{x+y+z}{xyz}=\dfrac{1}{2}(\dfrac{1}{xy}+\dfrac{1}{yz}+\dfrac{1}{xz})=VP $

Bài viết đã được chỉnh sửa nội dung bởi Hero Math: 03-07-2009 - 20:06


#29
cvp

cvp

    Sĩ quan

  • Thành viên
  • 400 Bài viết
Hãy tiếp tục nào:
Bài 16:Cho $a,b,c$ dương thỏa mãn $a^2+b^2+c^2=1$Chứng minh rằng:
$\dfrac{1}{1-bc}+\dfrac{1}{1-ca}+\dfrac{1}{1-ab}\le \dfrac{9}{2}$
Bài 17:Cho $a,b,c$ dương thỏa mãn $a^2+b^2+c^2=1$Chứng minh rằng:
$a^2\sqrt{1-bc}+b^2\sqrt{1-ca}+c^2\sqrt{1-ab}\ge \sqrt{\dfrac{2}{3}}$

p/s: Hai bài toán có liên hệ đó!

Hình đã gửi


#30
- Nguyên Lê -

- Nguyên Lê -

    Hạ sĩ

  • Thành viên
  • 54 Bài viết

Bài 16:Cho $a,b,c$ dương thỏa mãn $a^2+b^2+c^2=1$Chứng minh rằng:
$\dfrac{1}{1-bc}+\dfrac{1}{1-ca}+\dfrac{1}{1-ab}\le \dfrac{9}{2}$
Bài 17:Cho $a,b,c$ dương thỏa mãn $a^2+b^2+c^2=1$Chứng minh rằng:
$a^2\sqrt{1-bc}+b^2\sqrt{1-ca}+c^2\sqrt{1-ab}\ge \sqrt{\dfrac{2}{3}}$

$\Leftrightarrow\dfrac{bc}{1-bc}+\dfrac{ca}{1-ca}+\dfrac{ab}{1-ab}\le\dfrac32$
Bài 16: Áp dụng bất đẳng thức AM - GM và Schwarz:
$\dfrac{bc}{1-bc}\le\dfrac{(b+c)^2}{4-2(b^2+c^2)}=\dfrac12.\dfrac{(b+c)^2}{2a^2+b^2+c^2}\le\dfrac12\left(\dfrac{b^2}{a^2+b^2}+\dfrac{c^2}{a^2+c^2}\right)$

Tương tự cộng lại được đpcm

Bài 17: Tương tự:
$a^2\sqrt{1-bc}=\dfrac{a^2}{\sqrt2}.\sqrt{2-2bc}\ge\dfrac{a^2}{\sqrt2}\sqrt{1+a^2+b^2+c^2-b^2-c^2}=\dfrac{a^2}{\sqrt2}.\sqrt{1+a^2}$
Áp dụng CBS ta có:
$\\\left(\dfrac13+1\right)(a^2+1)\ge\left(\dfrac a{\sqrt3}+1\right)^2\\\Rightarrow\sqrt{a^2+1}\ge\dfrac{a+\sqrt3}2\\\Rightarrow\dfrac{a^2}{\sqrt2}.\sqrt{1+a^2}\ge\dfrac{a^2(a+\sqrt3)}{2\sqrt2}$

Tương tự cộng lại, kết hợp với $a^3+b^3+c^3\ge\dfrac1{\sqrt3}$ nữa là được. =.=
(Hình như cách hơi lằng nhằng)

Bài viết đã được chỉnh sửa nội dung bởi - Nguyên Lê -: 04-07-2009 - 23:50


#31
AvidAbel_9x09

AvidAbel_9x09

    Hạ sĩ

  • Thành viên
  • 50 Bài viết
Hôm trc em thấy ở Quyên ôn thi đại học của ông anh có bài này cũng hay phết!:
"cho ba số ko âm a,b,c thỏa mãn: $ a+b+c=1$. Tìm GTLN của biểu thức: $ A= a^{2}b+b^{2}c+c^{2}a $"
Em nghĩ đây là một bài hay nên mời các tiền bối cứ làm càng nhìu cách càng tốt giúp em! :)

#32
cvp

cvp

    Sĩ quan

  • Thành viên
  • 400 Bài viết
Lời giải của bạn ổn rùi.Mình xin đóng góp cách khác cho bài 2 để thấy sự liên hệ:
Áp dụng bđt Trê bư sép:
$a^2\sqrt{1-bc}+b^2\sqrt{1-ca}+c^2\sqrt{1-ab}\ge \dfrac{1}{3}(a^2+b^2+c^2)(\sqrt{1-bc}+\sqrt{1-ca}+\sqrt{1-ab})$
Vì vậy cần cm: $\sqrt{1-bc}+\sqrt{1-ca}+\sqrt{1-ab}\ge \sqrt 6$
Đến đây áp dụng CBS ta có $(\sqrt{1-bc}+\sqrt{1-ca}+\sqrt{1-ab})^2(\dfrac{1}{1-bc}+\dfrac{1}{1-ca}+\dfrac{1}{1-ab}\ge 27$
Sử dụng kết quả bài trên =>$\sqrt{1-bc}+\sqrt{1-ca}+\sqrt{1-ab}\ge \sqrt 6$
=>đpcm!
Dấu = khi $a=b=c=\dfrac{1}{\sqrt{3}}$

Hình đã gửi


#33
admireM

admireM

    Binh nhất

  • Thành viên
  • 49 Bài viết

Hôm trc em thấy ở Quyên ôn thi đại học của ông anh có bài này cũng hay phết!:
"cho ba số ko âm a,b,c thỏa mãn: $ a+b+c=1$. Tìm GTLN của biểu thức: $ A= a^{2}b+b^{2}c+c^{2}a $"
Em nghĩ đây là một bài hay nên mời các tiền bối cứ làm càng nhìu cách càng tốt giúp em! :wub:


Bài này khó làm sao mà lại thi đại học sao được, Bạn cứ nói là bài này bạn nghĩ ra cho nhanh:)

Giả sử a max, có các trường hợp sau:

a :wub: b :beat c

Khi đó $ a^2b+b^2c+ c^2a $ :ech $ (a+c)^2b $ :ech $4/27 [(a+c)/2 + (a+c)/2 +b]^3 $
=4/27

Trường hợp a :beat c :ech b khó chưa nghĩ ra đợi đấy.

Bài viết đã được chỉnh sửa nội dung bởi admireM: 05-07-2009 - 11:41


#34
admireM

admireM

    Binh nhất

  • Thành viên
  • 49 Bài viết

Bài này khó làm sao mà lại thi đại học sao được, Bạn cứ nói là bài này bạn nghĩ ra cho nhanh:)

Giả sử a max, có các trường hợp sau:

a :beat b :beat c

Khi đó A= $ a^2b+b^2c+ c^2a $ :ech $ (a+c)^2b $ :leq $4/27 [(a+c)/2 + (a+c)/2 +b]^3 $
=4/27

Trường hợp a :ech c :ech b khó chưa nghĩ ra đợi đấy.


Với trường hợp này:
A :leq $ (a+c/2)^2(b+c/2) $ :wub: $ 4/27[(a+c/2)/2+(a+c/2)/2 + (b+c/2)]^3 $ =4/27

Vậy A :wub: 4/27
Dấu bằng xảy ra khi a=2/3, b=1/3, c=0

Vậy Amax=4/27

Bài viết đã được chỉnh sửa nội dung bởi admireM: 05-07-2009 - 12:26


#35
admireM

admireM

    Binh nhất

  • Thành viên
  • 49 Bài viết
Ủng hộ một bài BDT hay (nhưng hơi khó):

Cho a, b, c là các số không âm thõa mãn: a+b+c=3
Chứng minh rằng $a^2+b^2+c^2+\dfrac{3}{2}abc$ :wub: $\dfrac{9}{2}$

Bài viết đã được chỉnh sửa nội dung bởi Nguyễn Hoàng Nam: 06-07-2009 - 17:52


#36
dungbo_213

dungbo_213

    Binh nhì

  • Thành viên
  • 14 Bài viết

Ủng hộ một bài BDT hay (nhưng hơi khó):

Cho a, b, c là các số không âm thõa mãn: a+b+c=3
Chứng minh rằng $a^2+b^2+c^2+(3/2)abc$ :wub: 9/2


biến đổi $p,q,r$

$2p^2-4q+3r \ge 9$
$9-4q+3r \ge 0$ (đúng với Schur )
Đường còn dài… và còn nhiều hơn chông gai…

Rồi thì lặng lẽ những tháng ngày buồn ở lại… ngày vui dễ lắng… mau phai…

Mai về nhìn lại cuộc đời vui ghê… về nhìn lại yêu thương vẫn thế… giữa cơn đau nặng nề… khốn

khó lê thê…

#37
cvp

cvp

    Sĩ quan

  • Thành viên
  • 400 Bài viết
Tiếp tục bài này:
Cho $a,b,c$ là ba cạnh của một tam giác thỏa mãn: $a^2+b^2+c^2=3$
Chứng minh rằng: $a+b+c\ge 2+abc$

Hình đã gửi


#38
Nguyễn Hoàng Nam

Nguyễn Hoàng Nam

    Độc thân...

  • Thành viên
  • 334 Bài viết
Chứng mình rằng:
$\dfrac{2}{b+c}+\dfrac{2}{a+b}+\dfrac{2}{a+c} \geq \dfrac{9}{a+b+c}$

Bài viết đã được chỉnh sửa nội dung bởi Nguyễn Hoàng Nam: 08-07-2009 - 09:55

Kho tư liệu bất đẳng thức

My blog

My website
Bán acc Megaupload giá rẻ, giảm giá đặc biệt cho các thành viên của VMF :D
Contact: 01644 036630

#39
dungbo_213

dungbo_213

    Binh nhì

  • Thành viên
  • 14 Bài viết

Chứng mình rằng:
$\dfrac{2}{b+c}+\dfrac{2}{a+b}+\dfrac{2}{a+c} \geq \dfrac{2}{a+b+c}$


đề sai ,vế phải là 9
sử dụng Svac
Đường còn dài… và còn nhiều hơn chông gai…

Rồi thì lặng lẽ những tháng ngày buồn ở lại… ngày vui dễ lắng… mau phai…

Mai về nhìn lại cuộc đời vui ghê… về nhìn lại yêu thương vẫn thế… giữa cơn đau nặng nề… khốn

khó lê thê…

#40
congcomMật khẩu:

congcomMật khẩu:

    Hạ sĩ

  • Thành viên
  • 84 Bài viết

Dùng giả thiết $ab+bc+ca=1$
BĐT $<=>3+\dfrac{c(a+b)}{ab}+\dfrac{b(c+a)}{ca}+\dfrac{a(b+c)}{bc} \ge 3+\dfrac{\sqrt{(a+b)(a+c)}}{a}+\dfrac{\sqrt{(b+a)(b+c)}}{b}+\dfrac{\sqrt{(c+a)(c+b)}}{c}$
Chứng minh $\dfrac{c(a+b)}{ab}+\dfrac{b(c+a)}{ca}+\dfrac{a(b+c)}{bc}\ge \dfrac{\sqrt{(a+b)(a+c)}}{a}+\dfrac{\sqrt{(b+a)(b+c)}}{b}+\dfrac{\sqrt{(c+a)(c+b)}}{c}$
Đến đây dùng AM-GM: $\dfrac{c(a+b)}{ab}+\dfrac{b(c+a)}{ca}\ge 2\dfrac{\sqrt{(a+b)(a+c)}}{a}$
Tương tự => đpcm
Dấu = khi $a=b=c=\dfrac{1}{\sqrt{3}}$

bài này nếu em nhớ ko nhầm thì ở trong toán học tuổi trẻ đúng ko hở heromath
cuộc đời ko bao giờ giữ lòng tự trọng cho bạn mà ban phải tự tạo ra nó




0 người đang xem chủ đề

0 thành viên, 0 khách, 0 thành viên ẩn danh